2
$\begingroup$

Fix an integer vector $\mathbf m\in \mathbb Z^k$. Let $q$ be a positive integer.

Is there a "good" upper bound in terms of $q,\bf m$ for the exponential sum: \[\sum_{\mathbf n} e\left(\frac{\langle m, n\rangle}{q}\right).\] Here $\langle \cdot, \cdot \rangle$ is usual inner product and $e(z) = e^{2\pi i z}$. The sum is taken over all ${\mathbf n=(n_1,\cdots,n_k)}$ such that $1\le n_i\le q$ and $gcd(n_1,\cdots,n_k,q)=1$.

When $k=1$, this is the famous Ramanujan sum. I have tried searching relevant results, but could not find any useful reference for $k\ge 2$. Any answers or references are appreciated!

$\endgroup$
4
  • $\begingroup$ Don't you need to assume some additional coprimality condition between each $n_i$ and $p$? For $k = 1$, for example, the usual Ramanujan sum includes the condition $(n,p) = 1$, for otherwise the sum is zero. $\endgroup$ Feb 7, 2017 at 0:54
  • $\begingroup$ Yes, you are right! I definitely forgot to add that. $\endgroup$
    – Changguang
    Feb 7, 2017 at 1:46
  • $\begingroup$ Doesn't $\sum e((m_1n_1+\cdots+m_kn_k)/p)$ just factor as $\sum e(m_1n_1/p)\times\cdots\times\sum e(m_kn_k/p)$? $\endgroup$ Feb 7, 2017 at 2:16
  • $\begingroup$ Yes, but the coprimality condition doesn't quite factor in a trivial way. $\endgroup$ Feb 7, 2017 at 2:16

1 Answer 1

3
$\begingroup$

Call this sum $c_q(m_1,\ldots,m_k)$. The indicator function of the condition $(n_1,\ldots,n_k,q) = 1$ can be written as the sum $\sum_{cd = (n_1,\ldots,n_k,q)} \mu(c)$, so that $d \mid q$, $c = \frac{q}{d}$, and $n_j \equiv 0 \pmod{c}$ for all $j \in \{1,\ldots,k\}$. It follows that $c_q(m_1,\ldots,m_k)$ may be written as \[\sum_{cd = (n_1, \ldots, n_k,q)} \mu(c) \prod_{j = 1}^{k} \sum_{n_j = 1}^{q} e\left(\frac{n_j m_j}{q}\right)= \sum_{d \mid q} \mu\left(\frac{q}{d}\right) \prod_{j = 1}^{k} \sum_{\substack{n_j = 1 \\ n_j \equiv 0 \pmod{\frac{q}{d}}}}^{q} e\left(\frac{n_j m_j}{q}\right). \] Writing $n_j = \frac{q}{d} \ell_j$, the inner sum is \[\sum_{\ell_j = 1}^{d} e\left(\frac{\ell_j m_j}{d}\right) = \begin{cases} d & \text{if $d \mid m_j$,} \\ 0 & \text{otherwise.} \end{cases}\] So \[c_q(m_1,\ldots,m_k) = \sum_{d \mid (m_1, \ldots, m_k, q)} \mu\left(\frac{q}{d}\right) d^k.\] This is multiplicative as a function of $q$, so that \[c_q(m_1,\ldots,m_k) = \prod_{p^r \parallel q} c_{p^r}(m_1,\ldots,m_k).\] So we need only determine $c_q(m_1,\ldots,m_k)$ when $q = p^r$ is a prime power. Since \[\mu(p^r) = \begin{cases} 1 & \text{if $r = 0$,} \\ -1 & \text{if $r = 1$,} \\ 0 & \text{if $r \geq 2$,} \end{cases}\] we have that for $q = p$, \[c_p(m_1,\ldots,m_k) = \begin{cases} p^k - 1 & \text{if $p \mid (m_1,\ldots,m_k)$,} \\ -1 & \text{if $p \nmid (m_1,\ldots,m_k)$,} \end{cases}\] while for $q = p^r$ with $r \geq 2$, \[c_{p^r}(m_1,\ldots,m_k) = \begin{cases} p^{(r - 1)k} (p^k - 1) & \text{if $p^r \mid (m_1,\ldots,m_k)$,} \\ -p^{(r - 1)k} & \text{if $p^{r - 1} \parallel (m_1,\ldots,m_k)$,} \\ -1 & \text{if $p^{r - 1} \nmid (m_1,\ldots,m_k)$.} \end{cases}\]

$\endgroup$
3
  • $\begingroup$ Peter, thank you very much for your answer. I think it's correct, but could you please double check your formulas? For example, line 4 with line 2; and last two formulas (not consistent). $\endgroup$
    – Changguang
    Feb 7, 2017 at 2:57
  • $\begingroup$ @Changguang, it should be fixed now. $\endgroup$ Feb 7, 2017 at 3:19
  • $\begingroup$ For the last two formulas, when $r=1$, I think that should be $p^k-1$, no? $\endgroup$
    – Changguang
    Feb 7, 2017 at 3:21

Your Answer

By clicking “Post Your Answer”, you agree to our terms of service and acknowledge you have read our privacy policy.

Not the answer you're looking for? Browse other questions tagged or ask your own question.